Đến nội dung

Hình ảnh

Topic về bất đẳng thức

* * * * * 16 Bình chọn

  • Please log in to reply
Chủ đề này có 206 trả lời

#101
Ispectorgadget

Ispectorgadget

    Nothing

  • Quản lý Toán Phổ thông
  • 2946 Bài viết
Cho $a;a1;...an;a1+a2+a3+...+an=2n$
Ta luôn có
$$\sum_{k=1}^{n}.\dfrac{a_{k}}{\sqrt[2]{a_{k+1}^{3}}}\geq \dfrac{2n}{3}Coia_{n+1}\equiv a_{1}b)\sum_{j=1}^{n}.\sum_{i=1}^{n}.\dfrac{x_{j}}{\sqrt{x_{i}^{3}+1}}\geq \dfrac{2n(n-1)}{3}$

Công thức bị lỗi nên xuất ra hơi xấu thông cảm
Cho em hỏi ở hàng thứ 3 mình dùng Côsi ngược dấu được không nếu ai biết thì post lời giải lên dùm

►|| The aim of life is self-development. To realize one's nature perfectly - that is what each of us is here for. ™ ♫


#102
alex_hoang

alex_hoang

    Thượng úy

  • Hiệp sỹ
  • 1152 Bài viết

Bài 53.

Cho $a,b,c >0$ và $a+b+c=1.$
Tìm max của:
$P=\dfrac{a^2+1}{b^2+1}+\dfrac{b^2+1}{c^2+1}+\dfrac{c^2+1}{a^2+1}$

Bài này quen thuộc đã được bàn nhiều trên diễn đàn
Nó vốn là bài toán trên tạp chí THTT với max tìm được là$\dfrac{7}{2}$
alex_hoang


HẸN NGÀY TRỞ LẠI VMF THÂN MẾN

http://www.scribd.co...oi-Ban-Cung-The

#103
alex_hoang

alex_hoang

    Thượng úy

  • Hiệp sỹ
  • 1152 Bài viết
Bài 54 Cho $x;\,y;\,z$ là các số thực dương thỏa mãn $xy + yz + zx = 3xyz$, chứng minh rằng:
$\dfrac{y^2}{xy^2+2z^2}+\dfrac{x^2}{zx^2+2y^2}+ \dfrac{z^2}{yz^2+2x^2}\ge 1$
Bài 55Cho bốn số thực $a,\;b,\;c,\;d$ thỏa mãn $a+b+c+d=6$và$a^2+b^2+c^2+d^2=12$ Chứng minh rằng
$36 \le 4(a^3+b^3+c^3+d^3)-(a^4+b^4+c^4+d^4) \le 48.$
Bài 56 Cho ba số thực dương $a;\,b;\,c$ có $abc=1$]. Tìm giá trị nhỏ nhất của
$P=\dfrac{a^2b}{a+b}+\dfrac{b^2c}{b+c}+\dfrac{c^2a}{c+a}$
Bài 57 Cho $a,b,c$ là các số thực dương, chứng minh rằng:
$a+b+c+ \dfrac{a^2}{b}+ \dfrac{b^2}{c}+ \dfrac{c^2}{a} \ge 6\left(\dfrac{a^2+b^2+c^2}{a+b+c}\right)$
P/s:Các bạn giải nốt bài 52 nữa nha
alex_hoang


HẸN NGÀY TRỞ LẠI VMF THÂN MẾN

http://www.scribd.co...oi-Ban-Cung-The

#104
Crystal

Crystal

    ANGRY BIRDS

  • Hiệp sỹ
  • 5534 Bài viết

Bài 57 Cho $a,b,c$ là các số thực dương, chứng minh rằng:
$$a+b+c+ \dfrac{a^2}{b}+ \dfrac{b^2}{c}+ \dfrac{c^2}{a} \ge 6\left(\dfrac{a^2+b^2+c^2}{a+b+c}\right)$$

Lời giải:
Không mất tính tổng quát, giả sử $a \le b \le c$. Khi đó bất đẳng thức được viết:
$$\sum {\left( {\dfrac{{{a^2}}}{b} + b - 2a} \right)} \ge 6\left( {\dfrac{{{a^2} + {b^2} + {c^2}}}{{a + b + c}}} \right) - 2\left( {a + b + c} \right) \Leftrightarrow \sum {\dfrac{{{{\left( {a - b} \right)}^2}}}{b} \ge } 6\left( {\dfrac{{{a^2} + {b^2} + {c^2}}}{{a + b + c}}} \right) - 2\left( {a + b + c} \right)$$
Áp dụng BĐT Cauchy - Schwarz, ta có:
$$\sum {\dfrac{{{{\left( {a - b} \right)}^2}}}{b} \ge } \dfrac{{{{\left( {\left( {a - b} \right) + \left( {b - c} \right) + \left( {a - c} \right)} \right)}^2}}}{{b + c + a}} = \dfrac{{4{{\left( {a - c} \right)}^2}}}{{a + b + c}}$$
Do đó ta cần chứng minh: $$2{\left( {a - c} \right)^2} \ge 3\left( {{a^2} + {b^2} + {c^2}} \right) - {\left( {a + b + c} \right)^2} \Leftrightarrow 2\left( {b - c} \right)\left( {b - a} \right) \le 0$$
Bất đẳng thức trên đúng do $a \le b \le c$. Vậy ta có đpcm. Đẳng thức xảy ra $ \Leftrightarrow a = b = c$.

#105
Crystal

Crystal

    ANGRY BIRDS

  • Hiệp sỹ
  • 5534 Bài viết

Bài 57 Cho $a,b,c$ là các số thực dương, chứng minh rằng:
$$a+b+c+ \dfrac{a^2}{b}+ \dfrac{b^2}{c}+ \dfrac{c^2}{a} \ge 6\left(\dfrac{a^2+b^2+c^2}{a+b+c}\right)$$

Một bài tương tự: Cho $a,b,c$ là các số thực dương, chứng minh rằng:
$$a + b + c + \dfrac{{{a^2}}}{b} + \dfrac{{{b^2}}}{c} + \dfrac{{{c^2}}}{a} \ge 2\sqrt {3\left( {{a^2} + {b^2} + {c^2}} \right)} $$
----------------------------------
Không dùng kết quả của bài toán 57.

#106
alex_hoang

alex_hoang

    Thượng úy

  • Hiệp sỹ
  • 1152 Bài viết
Bài 52

Do $abc=1$ nên bất đẳng thức trên có thể viết lại thành
$$1+a+b+c \ge 2\sqrt{1+ab+bc+ca}$$
Sử dụng bất đẳng thức AM-GM dạng $x+y \ge 2\sqrt{xy}$, ta có:
$$1+a+b+c \ge 2\sqrt{(1+a)(b+c)}$$
Vậy chứng minh sẽ hoàn tất nếu ta chỉ ra được
$$(1+a)(b+c) \ge 1+ab+bc+ca$$
Hay $$b+c \ge 1+bc$$
Tương đương với
$$(b-1)(c-1) \le 0$$
Mặt khác, do $abc=1$ nên trong ba số $a, b, c$, luôn tồn tại ít nhất một số lớn hơn $1$ và một số nhỏ hơn $1$.

Không mất tính tổng quát, ta có thể giả sử hai số đó là $b, c$ để từ đó suy ra $(b-1)(c-1) \le 0$.

Bài toán được giải quyết xong. Dấu đẳng thức xảy ra khi và chỉ khi $a=b=c=1$
alex_hoang


HẸN NGÀY TRỞ LẠI VMF THÂN MẾN

http://www.scribd.co...oi-Ban-Cung-The

#107
nguyenphu.manh

nguyenphu.manh

    Hạ sĩ

  • Thành viên
  • 89 Bài viết

Bài này quen thuộc đã được bàn nhiều trên diễn đàn
Nó vốn là bài toán trên tạp chí THTT với max tìm được là$\dfrac{7}{2}$

Thế anh đưa lời giải lên cho em biết với.
SLNA vô đối_pro


http://nghiloc2.forumvi.com

#108
DBSK

DBSK

    Binh nhất

  • Thành viên
  • 42 Bài viết
[

Bài 54 Cho $x;\,y;\,z$ là các số thực dương thỏa mãn $xy + yz + zx = 3xyz$, chứng minh rằng:
$\dfrac{y^2}{xy^2+2z^2}+\dfrac{x^2}{zx^2+2y^2}+ \dfrac{z^2}{yz^2+2x^2}\ge 1$
Bài 56 Cho ba số thực dương $a;\,b;\,c$ có $abc=1$]. Tìm giá trị nhỏ nhất của
$P=\dfrac{a^2b}{a+b}+\dfrac{b^2c}{b+c}+\dfrac{c^2a}{c+a}$

Bài 54:
Từ giả thiết $\rightarrow \dfrac{1}{x}+\dfrac{1}{y}+\dfrac{1}{z}=3$
Đặt $\dfrac{1}{x}=a,\dfrac{1}{y}=b,\dfrac{1}{z}=c \Rightarrow a+b+c=3$
BDT$ \Rightarrow \sum \dfrac{a^2}{a+2b^2}\ge 1$
$\Rightarrow \sum (a-\dfrac{2ab^2}{a+2b^2})\ge 1$
$\Leftrightarrow 3-\sum\dfrac{2ab^2}{a+2b^2}\ge 1$
Ta có:

$\dfrac{2ab^2}{a+2b^2}\le^{AM-GM} \dfrac{2}{3}\sqrt[3]{a^2b^2}$

Tương tự ta có:

$VT\ge 3-\dfrac{2}{3}(\sum\sqrt[3]{a^2b^2})$

Mà:

$\sum\sqrt[3]{a^2b^2}\le \sum\dfrac{ab+ab+1}{3}=\dfrac{2}{3}(ab+bc+ca)+1\le 3$
Vậy $VT \ge 3-2=1 (dpcm)$
Bài 56:
$ abc=1\to a=\dfrac{x}{y},b=\dfrac{y}{z},c=\dfrac{z}{x}$

${{x}^{4}}+{{y}^{4}}+{{z}^{4}}+3\left( {{x}^{2}}{{y}^{2}}+{{y}^{2}}{{z}^{2}}+{{z}^{2}}{{x}^{2}} \right)$
$={{\left( {{x}^{2}}+{{y}^{2}}+{{z}^{2}} \right)}^{2}}+\left( {{x}^{2}}{{y}^{2}}+{{y}^{2}}{{z}^{2}}+{{z}^{2}}{{x}^{2}} \right)\le \dfrac{4}{3}{{\left( {{x}^{2}}+{{y}^{2}}+{{z}^{2}} \right)}^{2}}$

$ P=\sum\limits_{cyc}{\dfrac{{{a}^{2}}b}{a+b}} = \sum\limits_{cyc}{\dfrac{\dfrac{{{x}^{2}}}{{{y}^{2}}}.\dfrac{y}{z}}{\dfrac{x}{y}+\dfrac{y}{z}}} = \sum\limits_{cyc}{\dfrac{2{{x}^{2}}}{2xz+2{{y}^{2}}}} \overset{AM-GM}{ \ge }\,2.\sum\limits_{cyc}{\dfrac{{{x}^{2}}}{{{x}^{2}}+{{z}^{2}}+2{{y}^{2}}}}$
$\overset{Cauchy-Schwarz}{ \ge }\,2.\dfrac{{{\left( {{x}^{2}}+{{y}^{2}}+{{z}^{2}} \right)}^{2}}}{{{x}^{4}}+{{y}^{4}}+{{z}^{4}} + 3\left( {{x}^{2}}{{y}^{2}}+{{y}^{2}}{{z}^{2}}+{{z}^{2}}{{x}^{2}} \right)}\ge 2.\dfrac{3}{4}=\dfrac{3}{2}$

Từ đó: $P=\sum\limits_{cyc}{\dfrac{{{a}^{2}}b}{a+b}}\ge \dfrac{3}{2}$

Bài viết đã được chỉnh sửa nội dung bởi DBSK: 08-11-2011 - 09:41


#109
Ispectorgadget

Ispectorgadget

    Nothing

  • Quản lý Toán Phổ thông
  • 2946 Bài viết
Bài 63 tương tự bài toán ở đây http://diendantoanho...l=&fromsearch=1

►|| The aim of life is self-development. To realize one's nature perfectly - that is what each of us is here for. ™ ♫


#110
DBSK

DBSK

    Binh nhất

  • Thành viên
  • 42 Bài viết
Mong bạn Hoàng không nên lấy bài từ boxmath cop sang đây!
Làm thế thật không hay mình thấy bên đó có bài nào là bạn lại post sang đây!
Gây ra sự trùng lặp giưa các trang web!

#111
alex_hoang

alex_hoang

    Thượng úy

  • Hiệp sỹ
  • 1152 Bài viết
Vài ý muốn nói với bạn DBSK
Có thật là bên ấy có bài nào thì mình đều copy và post sang đây hay không bạn nên chú ý lời nói và theo ý kiên riêng tôi việc đi tìm tòi những bài toán hay của diễn đàn khác để làm phong phú cho diendantoanhoc hơn thì chẳng có gì sai cả và thực tế nhiều diễn đàn khác cũng vậy , có thể bạn biết nhưng nhiều người khác không biết có thể bạn làm rồi nhưng nhiều người khác chưa làm.Tuy nhiên tôi sẽ xóa bài viết trên và dù sao cũng cảm ơn ý kiến của bạn
Bài 58:Cho các số thực dương $a,b,c$ sao cho $abc=1$.Tìm hằng số $k$ lớn nhất để bất đẳng thức sau đúng
\[(a + b)(b + c)(c + a) + k \ge \dfrac{{8 + k}}{3}(a + b + c)\]

Bài viết đã được chỉnh sửa nội dung bởi alex_hoang: 10-11-2011 - 20:42

alex_hoang


HẸN NGÀY TRỞ LẠI VMF THÂN MẾN

http://www.scribd.co...oi-Ban-Cung-The

#112
LilTee

LilTee

    Binh nhất

  • Thành viên
  • 26 Bài viết
Lời giải bài 56 mình khẳng định là copy và paste lời giải bên boxmath.vn.
Bài 57
Cho $a,b,c$ là các số thực dương thoả mãn $a^3+b^3+c^3=3$. Chứng minh rằng

$$\dfrac{ab}{\sqrt[3]{{{b}^{3}}+8}}+\dfrac{bc}{\sqrt[3]{{{c}^{3}}+8}}+\dfrac{ca}{\sqrt[3]{{{a}^{3}}+8}}\le \dfrac{\sqrt{{{a}^{2}}+{{b}^{2}}+{{c}^{2}}}}{\sqrt[6]{3}}$$

Diễn đàn Vật lí phổ thông: https://vatliphothong.vn
My Blog: http://tanghaituan.com

Học trực tuyến: https://hoctructuyen.tv


#113
alex_hoang

alex_hoang

    Thượng úy

  • Hiệp sỹ
  • 1152 Bài viết
Cảm ơn anh rất nhiều em thất sự em cũng muốn diễn đàn này trao đổi một số đề bài bên bothmath nhưng không ngờ bạn DBSK lại copy paste lời giải bên bothmath
Dù sao cũng là lỗi tại em đã lấy bài bên bothmath trước mới đẫn đến bạn trích lời giải paste sang.Em xin tạ tội với mọi người và hứa là không bao giờ lấy bài bên ấy về đây nữa.
Cảm ơn anh Huân nhiều
alex_hoang


HẸN NGÀY TRỞ LẠI VMF THÂN MẾN

http://www.scribd.co...oi-Ban-Cung-The

#114
taminhhoang10a1

taminhhoang10a1

    Trung sĩ

  • Thành viên
  • 131 Bài viết

Mong bạn Hoàng không nên lấy bài từ boxmath cop sang đây!
Làm thế thật không hay mình thấy bên đó có bài nào là bạn lại post sang đây!
Gây ra sự trùng lặp giưa các trang web!

Theo em thì lấy cũng càng tốt cho diễn đàn mình chứ sao. Thử hỏi bây giờ có ai đó lấy bài ở diễn đàn của nước ngoài toàn tiếng Anh về dịch ra VIệt cũng cấm người ta à. Mà như bọn em thường hay vao diendantoanhoc.net hơn la boxmath.vn thì việc anh Hoàng lấy bài từ bên đó cho chúng em làm là hợp lý. Biết đâu bên này có cách giải hay hơn bên kia thì sao đây. DBSK nên xem lại lời nói có phần xúc phạm này. Thành viên của 2 diễn đàn nên trao đổi bài với nhau chứ

Bài viết đã được chỉnh sửa nội dung bởi taminhhoang10a1: 11-11-2011 - 16:54

THPT THÁI NINH - THÁI THỤY - THÁI BÌNH

#115
Zaraki

Zaraki

    PQT

  • Phó Quản lý Toán Cao cấp
  • 4273 Bài viết
http://diendantoanho...showtopic=36201
Trên đây là link một số bài BĐT tự sáng tác của anh Dương Đức Lâm.

Discovery is a child’s privilege. I mean the small child, the child who is not afraid to be wrong, to look silly, to not be serious, and to act differently from everyone else. He is also not afraid that the things he is interested in are in bad taste or turn out to be different from his expectations, from what they should be, or rather he is not afraid of what they actually are. He ignores the silent and flawless consensus that is part of the air we breathe – the consensus of all the people who are, or are reputed to be, reasonable.

 

Grothendieck, Récoltes et Semailles (“Crops and Seeds”). 


#116
LilTee

LilTee

    Binh nhất

  • Thành viên
  • 26 Bài viết

Cảm ơn anh rất nhiều em thất sự em cũng muốn diễn đàn này trao đổi một số đề bài bên bothmath nhưng không ngờ bạn DBSK lại copy paste lời giải bên bothmath
Dù sao cũng là lỗi tại em đã lấy bài bên bothmath trước mới đẫn đến bạn trích lời giải paste sang.Em xin tạ tội với mọi người và hứa là không bao giờ lấy bài bên ấy về đây nữa.
Cảm ơn anh Huân nhiều

Anh tên Tuân em nha :D.
Anh là Smod bên boxmath, thay mặt bên đó anh muốn nói là em cứ thoải mái lấy bài bên đó và post sang đây cho mọi người cùng giải, cùng bàn luận. Vì các diễn đàn post bài của nhau để trao đổi là chuyện bình thường mà em. Nhưng khi lấy thì em nên trích dẫn nguồn là xong thôi mà, không phải đến nỗi là "hứa là không bao giờ lấy bài bên ấy về đây nữa." đâu em nha :).
Còn về DBSK thì cũng nên trích dẫn lời giải của ai chứ đừng copy rồi paste nguyên si như vậy là được.
Thế nhé!
Thân,
Lil.Tee

Diễn đàn Vật lí phổ thông: https://vatliphothong.vn
My Blog: http://tanghaituan.com

Học trực tuyến: https://hoctructuyen.tv


#117
alex_hoang

alex_hoang

    Thượng úy

  • Hiệp sỹ
  • 1152 Bài viết
Ngại mặt kinh đến tên anh em cũng nhớ sai thế mới chết chứ
Bài 57 của anh em làm thế này đây
Sử dụng bất đẳng thức Hoder ta có
\[VT \le \sqrt[3]{{{{\left( {a\sqrt a + b\sqrt b + c\sqrt c } \right)}^2}(\dfrac{{{b^3}}}{{{b^3} + 8}} + \dfrac{{{c^3}}}{{{c^3} + 8}} + \dfrac{{{a^3}}}{{{a^3} + 8}})}}\]
Ta chứng minh
\[\dfrac{{{b^3}}}{{{b^3} + 8}} + \dfrac{{{c^3}}}{{{c^3} + 8}} + \dfrac{{{a^3}}}{{{a^3} + 8}} \le \dfrac{1}{3} \Leftrightarrow \dfrac{8}{{{b^3} + 8}} + \dfrac{8}{{{c^3} + 8}} + \dfrac{8}{{{c^3} + 8}} \ge \dfrac{8}{9}\]
Bất đẳng thức này hiển nhiên đúng theo Cauchy Schwarz
Và\[a\sqrt a + b\sqrt b + c\sqrt c \le \sqrt {\left( {{a^2} + {b^2} + {c^2}} \right)(a + b + c)} \]
alex_hoang


HẸN NGÀY TRỞ LẠI VMF THÂN MẾN

http://www.scribd.co...oi-Ban-Cung-The

#118
alex_hoang

alex_hoang

    Thượng úy

  • Hiệp sỹ
  • 1152 Bài viết
Đây là các BĐT của anh DĐLâm mời các bạn nhào vô




Trước khi offline 1 thời gian,mình muốn gửi đến DD những BDT mới nhất-hầu hết đều là sáng tác.Hy vọng sẽ hâm nóng lại topic BDT thời gian qua hơi im lìm.
Bài 1: Cho $a,b,c\geq0$.CMR:
$\dfrac{ab}{c}+\dfrac{bc}{a}+\dfrac{ca}{b}+a+b+c\geq\sqrt{2(a^2+b^2)}+\sqrt{2(b^2+c^2)}+\sqrt{2(c^2+a^2)}$

Bài 2: Cho $a,b,c\geq0$.CMR:
$\sqrt{3(a^4+b^4+c^4)}+ab+bc+ca \geq \dfrac2{3}(a+b+c)^2$

Bài 3:Cho $a,b,c\geq0$.CMR:
$\sqrt{a^4+b^4+c^4}+\dfrac{abc}{a+b+c} \geq (\dfrac1{3}+\sqrt3)\sqrt[3]{a^2b^2c^2}$

Bài 4:Cho $a,b,c\geq0$.CMR:
$\dfrac1{5a^2+bc}+\dfrac1{5b^2+ca}+\dfrac1{5c^2+ab} \geq \dfrac3{a^2+b^2+c^2+ab+bc+ca}$

Bài 5: Cho $a,b,c,d,e \geq0$.CMR:
a) $\sqrt[4]{\dfrac{a^4+b^4+c^4+d^4}4} \geq \sqrt[3]{\dfrac{abc+bcd+cda+dab}4}$
b)$\dfrac{a+b+c+d+e}5\geq\sqrt{\dfrac{ab+ac+ad+ae+bc+bd+be+cd+ce+de}{10}}$

Bài 6:Cho $a,b,c\geq1$.CMR:
$(1-a^3)(1-b^3)(1-c^3) \geq(1-abc)^3$
TQ: Có hay ko BĐT sau với mọi $a_i\geq1 $và$ 3\leq n \in N$
$ (1-a_1^n)(1-a_2^n)...(1-a_n^n) \geq(1-a_1a_2...a_n)^n$

Bài 7:Cho $a,b,c\geq0$.CMR:
$ (1+a^5)(1+b^5)(1+c^5)\geq (1+a^3b^2)(1+b^3c^2)(1+c^3a^2)$

Bài 8:Cho $a,b,c,d\geq0$.CMR:
$a^4+b^4+c^4+12abcd\geq(a+b+c+d)(abc+bcd+cda+dab)$

Bài 9:[Trần Tuấn Anh]Cho $a,b,c\geq0$.CMR:
$\dfrac{a^3}{a^2+ab+b^2}+\dfrac{b^3}{b^2+bc+c^2}+\dfrac{c^3}{c^2+ca+a^2}$$\geq\dfrac{\sqrt[3]{4(a^3+b^3)} +\sqrt[3]{4(b^3+c^3)}+\sqrt[3]{4(c^3+a^3)}}6$

Bài 10:[Trần Tuấn Anh]Tìm số thực k nhỏ nhất sao cho với mọi $a,b,c$ thỏa mãn $min{(a,b,c)}\geq k$ thì
$a^3+b^3+c^3+3 \geq ab+bc+ca+a+b+c$

Bài 11:Cho a,b,c đôi một khác nhau.CMR:
$ \dfrac1{(a-b)^2}+ \dfrac1{(b-c)^2}+ \dfrac1{(c-a)^2} \geq \dfrac9{4(a^2+b^2+c^2+ab+bc+ca)}$

Bài 12:Cho $a,b,c \in[0,1$].CMR:
$\dfrac{a^3-abc}{(1+a)^3}+\dfrac{b^3-abc}{(1+b)^3}+\dfrac{c^3-abc}{(1+c)^3} \geq 0$
........


$k=-1$

Bài 13:Cho $a,b,c\ge0:abc\ge1.$ Tìm Max:

$P=\dfrac{(1+a)^4+(1+b)^4+(1+c)^4}{a^3(1+a^2)+b^3(1+b^2)+c^3(1+c^2)}$

Bài 14: Cho $a_i \in R,i=1,2...n,2 \le n \in N$.Tìm Min:

$P= \sum\limits_{i=1}^{n} a_i^2 +(\dfrac{ \sum\limits_{i \neq j}^{n}a_ia_j }{ \sum\limits_{i=1}^{n}a_i })^2$

Bài 15: Cho $a,b,c\ge0:(a+b-c)(b+c-a)(c+a-b)=8$.Tìm Max:
$ P=\sum \dfrac1{\sqrt[3]{1+a}}$

Bài 16: Cho $a,b,c\ge 1:a+b+c+abc=ab+bc+ca+2$.CM:
$ \dfrac a{b}+ \dfrac b{c}+ \dfrac c{a}+ \dfrac {4}{abc}\ge 2(1+ \dfrac 1{ab}+ \dfrac 1{bc}+ \dfrac 1{ca} )$

Bài 17: Cho $a,b,c\ge0:abc=1$. Cm:
$a+b+c\ge \sum\dfrac{1+\sqrt2}{1+\sqrt2a}$

Bài 18: Cho $a,b,c\ge 0:abc=1$.CM
$ a+b+c \ge \sqrt{\sqrt a+\sqrt b+\sqrt c}$
....(còn nữa).....


Sử dụng BĐT cổ điển luôn là PP ấn tượng nhất !


Bài 19: Cho $a,b,c\ge0:ab+bc+ca\le3$. CM:
$\dfrac1{7+a^2}+\dfrac1{7+b^2}+\dfrac1{7+c^2}\le \dfrac2{(2+a+b)^2}+\dfrac2{(2+b+c)^2}+\dfrac2{(2+a+c)^2}$

Bài 20:Cho $a,b,c$ ko âm có tổng bằng 3. Tìm max :
$P=a^3b+b^3c+c^3a$

Bài 21:Cho $a,b,c$ ko âm.CM:
$\sum_{cyc} \dfrac{a^2}{a+b}\ge \sum_{cyc} \sqrt{\dfrac{a^2+b^2}2}$

Bài 22:Cho $a,b,c$ ko âm.CM:
$\sum_{cyc}\dfrac{a^3}{a^2-ab+b^2} \le \sum_{cyc}\sqrt{\dfrac{a^3+b^3}2}$

...(continue)...




Sử dụng BĐT cổ điển luôn là PP ấn tượng nhất !




alex_hoang


HẸN NGÀY TRỞ LẠI VMF THÂN MẾN

http://www.scribd.co...oi-Ban-Cung-The

#119
LilTee

LilTee

    Binh nhất

  • Thành viên
  • 26 Bài viết

Ngại mặt kinh đến tên anh em cũng nhớ sai thế mới chết chứ
Bài 57 của anh em làm thế này đây
Sử dụng bất đẳng thức Hoder ta có
\[VT \le \sqrt[3]{{{{\left( {a\sqrt a + b\sqrt b + c\sqrt c } \right)}^2}(\dfrac{{{b^3}}}{{{b^3} + 8}} + \dfrac{{{c^3}}}{{{c^3} + 8}} + \dfrac{{{a^3}}}{{{a^3} + 8}})}}\]
Ta chứng minh
\[\dfrac{{{b^3}}}{{{b^3} + 8}} + \dfrac{{{c^3}}}{{{c^3} + 8}} + \dfrac{{{a^3}}}{{{a^3} + 8}} \le \dfrac{1}{3} \Leftrightarrow \dfrac{8}{{{b^3} + 8}} + \dfrac{8}{{{c^3} + 8}} + \dfrac{8}{{{c^3} + 8}} \ge \dfrac{8}{9}\]
Bất đẳng thức này hiển nhiên đúng theo Cauchy Schwarz
Và\[a\sqrt a + b\sqrt b + c\sqrt c \le \sqrt {\left( {{a^2} + {b^2} + {c^2}} \right)(a + b + c)} \]

Bài giải của em rất hay, ngoài suy nghĩ của anh khi tạo ra bài toán này. Thanks

Diễn đàn Vật lí phổ thông: https://vatliphothong.vn
My Blog: http://tanghaituan.com

Học trực tuyến: https://hoctructuyen.tv


#120
alex_hoang

alex_hoang

    Thượng úy

  • Hiệp sỹ
  • 1152 Bài viết
Ngoài suy nghĩ tức là anh còn cách khác anh có thể post lên cho mọi người tham khảo không
alex_hoang


HẸN NGÀY TRỞ LẠI VMF THÂN MẾN

http://www.scribd.co...oi-Ban-Cung-The




1 người đang xem chủ đề

0 thành viên, 1 khách, 0 thành viên ẩn danh